ex 1. 32(29)-24

Réussis tes devoirs et examens dès maintenant avec Quizwiz!

hemophilia B (

Christmas disease, factor IX deficiency

Multiple myeloma, or plasma cell myeloma

Clinical Manifestations -Bone degeneration causes calcium loss from bones, resulting in hypercalcemia and causing renal, gastrointestinal (GI), or neurological changes, such as polyuria, anorexia, confusion, and ultimately seizures, coma, and cardiac problems. -Develops slowly and insidiously. -Asymptomatic until advanced disease state. -Skeletal pain is the major symptom: pelvis, spine, and ribs are common sites of pain. -Osteolytic lesions are seen on radiographs of the skull, vertebrae, and ribs.

Stage I: involvement of single lymph node Stage II: involvement of two or more lymph nodes on one side of the diaphragm Stage III: lymph node involvement above and below the diaphragm Stage IV: involvement outside of diaphragm (other organ systems) Stage A (absence) or B (presence) indicates significant systemic symptoms (e.g., fever, night sweats, weight loss).

Collaborative care for Hodgkin's lymphoma pertains to choosing an appropriate regimen based on the staging determined by diagnostic studies. Staging depends on the degree or severity of the disease in the body and the presence of B symptoms. The nomenclature used in staging involves an A or B classification, depending on whether symptoms are present when the disease is found, and a Roman numeral (I to IV) that reflects the location and extent of the disease.

Stomatitis, Mucositis, and Esophagitis

Encourage patient to keep oral cavity clean and moist by frequent oral rinses with saline or salt and soda solution. A saline solution of 1 tsp of salt in 1 L of water is an effective cleansing agent;

Hodgkin's Lymphoma CLINICAL MANIFESTATION

Enlargement of the cervical, axillary, or inguinal lymph nodes. Not painful unless pressure is exerted on adjacent nerves. Weight loss, fatigue, weakness, fever, chills, tachycardia, or night sweats. Initial findings of fever, night sweats, and weight loss (referred to as B symptoms) correlate with a poorer prognosis. Generalized pruritus without skin lesions. Cough, dyspnea, stridor, and dysphagia may all reflect mediastinal node involvement.

Hodgkin's Lymphoma Overview

Etiology is unknown. Risk factors: Epstein-Barr virus infection (also referred to as mononucleosis), genetic predisposition, and exposure to occupational toxins. Disease arises at a single location and then spreads along adjacent lymphatics (In 80% of patients, disease originates in the cervical lymph nodes). Eventually infiltrates other organs, especially the lungs, spleen, and liver. Lymph node structure is destroyed by hyperplasia of monocytes and macrophages.

Thrombocytopenia Inherited Caused by:

Fanconi anemia (pancytopenia) Hereditary thrombocytopenia

Collaborative care of HIT includes:

Heparin must be discontinued when HIT is first recognized to halt platelet destruction and reduce vascular endothelial injury. Heparin flushes for vascular catheters should also be stopped. To maintain anticoagulation, the patient should be started on a direct thrombin inhibitor, such as argatroban. Fondaparinux, a factor Xa inhibitor (indirect thrombin inhibitor), may be used. Warfarin should be started only when the platelet count has reached 150,000/µL. If the clotting is severe, the most commonly used treatment modalities are plasmapheresis to clear the platelet-aggregating IgG from the blood, protamine sulfate to interrupt the circulating heparin, thrombolytic agents to treat the thromboembolic events, and surgery to remove clots. Platelet transfusions are not effective because they may enhance thromboembolic events. Patients who have had HIT should never be given heparin or low-molecular-weight heparin (LMWH) as HIT is often caused by an immune reaction to heparin. This should be clearly marked on the patient's medical record.

idiopathic thrombocytopenic purpura (ITP)

ITP is the most common acquired thrombocytopenia. It is a syndrome of abnormal destruction of circulating platelets. ITP is an autoimmune disease. In ITP, platelets are coated with antibodies. Although these platelets function normally, when they reach the spleen the antibody-coated platelets are recognized as foreign and are destroyed by macrophages. Platelets normally survive 8-10 days, but in ITP survival is shortened. Chronic ITP occurs most commonly in women 15-40 years of age. Chronic ITP has a gradual onset and transient remissions occur.

HODGKINS LYMPHOMA-

Originates in the cervical lymph nodes in 60-70% of patients. Mediastinal lymph nodes are second most common.

TTP may be treated in a variety of ways:

Plasma exchange (plasmapheresis) may be needed to aggressively reverse the process. Treatment should be continued daily until the patient's platelet counts normalize and hemolysis has ceased. Corticosteroids may be added to this treatment. Rituximab has been used for patients who are refractory to plasma exchange. Other immunosuppressants, such as cyclosporine or cyclophosphamide, may also be used. Splenectomy may be considered in patients who are refractory to plasma exchange or immunosuppression. Administration of platelets is generally contraindicated because it may lead to new vWF-platelet complexes and increased clotting.

ACUTE LYMPHOCYTIC LEUKEMIA (ALL)

Presence of Philadelphia chromosome in 20%-25% of patients

The nurse is reviewing a patient's chart and the patient is found to have a decreased platelet count (60,000/µL). How can the nurse determine the type of thrombocytopenia the patient is experiencing? · Vital signs · Radiology reports · Physical examination · Prior medical history

Prior medical history Heparin-induced thrombocytopenia (HIT) occurs in response to heparin administration, and thrombotic thrombocytopenic purpura (TTP) may follow drug toxicity, pregnancy or preeclampsia, infection, or known autoimmune disorder. Therefore, the patient history can help differentiate the type of thrombocytopenia.

A number of disorders are known to predispose patients to DIC, including the following:

Shock Sepsis Placental abruption Acute liver failure Malignancies Severe head injury Heatstroke Massive tissue trauma Crush injuries Pulmonary emboli

Thrombocytopenia Acquired-Nonimmune

Shortened circulation (increased consumption) Thrombotic thrombocytopenic purpura (TTP) Disseminated intravascular coagulation (DIC) Heparin-induced thrombocytopenia (HIT) Splenomegaly or splenic sequestration Turbulent blood flow (hemangiomas, abnormal cardiac valves, intraaortic balloon pumps) Decreased production Drug-induced marrow suppression Chemotherapy Viral infection [hepatitis C (HCV), human immunodeficiency virus (HIV), cytomegalovirus (CMV)] Bacterial infection (sepsis) Alcoholism, bone marrow suppression Myelodysplastic syndrome (MDS) Myelofibrosis Aplastic anemia Hematologic malignancy (leukemias, lymphomas, myeloma) Solid tumor infiltrating bone marrow Radiation to the bone

thrombotic thrombocytopenic purpura (TTP)

TTP is an uncommon syndrome characterized by hemolytic anemia, thrombocytopenia, neurologic abnormalities, fever (in the absence of infection), and renal abnormalities. In most cases, the syndrome is caused by the deficiency of a plasma enzyme (ADAMTS13) that breaks the von Willebrand (vWF) clotting factor into normal size. If the ADAMTS13 plasma enzyme is not present in the body, then the von Willebrand clotting factors become too large and attach to activated platelets, resulting in enhanced platelet aggregation. The syndrome may be idiopathic or caused by drug toxicity, pregnancy or preeclampsia, infection, or known autoimmune disorder. TTP is a medical emergency because bleeding and clotting occur simultaneously.

TTP may be treated in a variety of ways:

The first step is to treat the underlying disorder (e.g., infection) or remove the causative agent (such as a drug), if identified. If untreated, TTP usually results in irreversible renal failure and death.

Many patients with thrombocytopenia are usually asymptomatic. The most common symptom is bleeding, usually mucosal or cutaneous (seen in ITP).

The most common symptom is bleeding, usually mucosal or cutaneous (seen in ITP). . -Mucosal bleeding may manifest as epistaxis and gingival bleeding, and large bullous hemorrhages may appear on the buccal mucosa. -Bleeding into the skin is manifested as petechiae, purpura, or superficial ecchymosis. -Prolonged bleeding after routine procedures such as venipuncture or intramuscular (IM) injection may indicate thrombocytopenia. -Because bleeding may be internal, be aware of manifestations that reflect this type of blood loss, including weakness, fainting, dizziness, tachycardia, abdominal pain, and hypotension. -Other disorders, such as thrombotic thrombocytopenic purpura (TTP) and heparin-induced thrombocytopenia (HIT), can cause severe clotting and resultant complications throughout bodily systems. The major complication of thrombocytopenia is hemorrhage. It may occur in any area of the body, including the joints, retina, and brain. Cerebral hemorrhage may be fatal.

Nursing care for patients with Hodgkin's lymphoma is primarily based on managing symptoms related to the disease, such as pain, and side effects of therapy, such as pancytopenia.

The physical, psychological, social, and spiritual consequences of the patient's disease must be addressed.

ACUTE LYMPHOCYTIC LEUKEMIA (ALL)

The presence of an excessive number of immature small lymphocytes in the bone marrow . most frequently seen in children with a mean age of 15 years at diagnosis · Patients with ALL have a HIGH probability of having leukemic infiltration of CNS p636

Thrombocytopenia Acquired-Drug and Herbal Causes

Thiazide diuretics Alcohol Chemotherapeutic drugs Digoxin Nonsteroidal anti-inflammatory drugs (NSAIDs): ibuprofen, indomethacin, naproxen Antibiotics: penicillins, cephalosporins, sulfonamides Other antiinfectives: rifampin, ganciclovir, amphotericin B Analgesics: aspirin and aspirin-containing drugs, acetaminophen Antipsychotic and antiseizure agents: haloperidol, valproate, lithium Platelet glycoprotein inhibitors: abciximab, tirofiban, eptifibatide, clopidogrel H2-receptor antagonists: cimetidine, ranitidine Gold compounds: auranofin Spices: ginger, cumin, turmeric, cloves Vitamins: vitamin C, vitamin E Heparin Herbs: angelica, bilberry, feverfew, garlic, ginkgo biloba, ginseng Quinine compounds: tonic water

THALIDOMIDE-

Treatment for multiple myeloma

Multiple myeloma, or plasma cell myeloma, is

a condition in which neoplastic plasma cells infiltrate the bone marrow and destroy bone. . It is twice as common in men as in women and usually develops after age 40. Although considered incurable, the 5-year survival rate is close to 45% because of the variety of treatments available throughout the course of the disease. . Etiology Etiology is unknown. Risk factors: exposure to radiation, organic chemicals (e.g., benzene), herbicides, and insecticides. Other factors: obesity, genetic factors, and viral infection.

• Thrombocytopenia is

a reduction of platelets below 150,000/μL (150 × 109 /L). • Most platelet disorders are acquired. A common cause is the ingestion of certain drugs. • They can be categorized by impaired production, increased destruction, and abnormal distribution. • The most common acquired thrombocytopenia is a syndrome of abnormal destruction and reduced production of circulating platelets termed immune thrombocytopenic purpura (ITP).

Sickle cell crisis is p617

a severe, painful, acute exacerbation of RBC sickling, causing a vaso-occlusive crisis.

Thrombocytopenia is a condition characterized by

abnormally low levels of platelets, also known as thrombocytes, in the blood. A normal human platelet count ranges from 150,000 to 450,000 platelets per microliter of blood.

· Microangiopathic

disseminated intravascular coagulation (DIC), thrombotic thrombocytopenic purpura (TTP)

Aspects of nursing management for patients with multiple myeloma are different. Specific management includes

hydration, prevention of pathological fractures, pain control, and addressing other ancillary concerns.

ACUTE MYELOGENOUS LEUKEMIA (AML)

increase in the number of myeloblast . accounts for 15%-20% of acute leukemia in children and 80% in adults

Myelodysplastic syndrome (MDS) P634

is a group of related hematologic disorders characterized by peripheral blood cytopenias (from ineffective blood cell production) and changes in the cellularity of the bone marrow with dysplastic changes. In MDS, hematopoiesis is disorderly and ineffective. The estimated incidence is 4.9 per 100,000 people per year in the United States. Although it can occur in all age-groups, the highest prevalence is in people over 80 years of age, at which point the rate increases to 59.8 per 100,000.

18.P634 myelodysplastic syndrome MDS-

is a group of related hematologic disorders characterized by peripheral blood cytopenias (from ineffective blood cell production) and changes in the cellularity of the bone marrow with dysplastic changes.Bone marrow biopsy with aspirate analysis is essential for both the diagnosis and classification of the specific type of MDS. Laboratory data and bone marrow studies help rule out other causes of the dysplasia, such as nonmalignant disorders, cobalamin and folate deficiencies, and infections.

Sickle cell crisis p617

is a severe, painful, acute exacerbation of RBC sickling, causing a vaso-occlusive crisis. As sickled cells impair blood flow, vasospasm occurs, further restricting blood flow. Severe capillary hypoxia causes changes in membrane permeability, leading to plasma loss, hemoconcentration, thrombi, and further circulatory stagnation. Tissue ischemia, infarction, and necrosis eventually occur from lack of O2. Shock is a possible life-threatening consequence of sickle cell crisis because of severe O2 depletion of the tissues and a reduction of the circulating fluid volume. Sickle cell crisis can begin suddenly and persist for days to weeks.

primary hemostasis- pg 622

is involves the severed vessel constricting and platelets collecting at the injury site. p623 Secondary hemostasis occurs when thromboplastin is released, prothrombin converts to thrombin, and fibrin is lysed.

Polycythemia

is the production and presence of increased numbers of RBCs. The increase in RBCs can be so great that blood circulation is impaired because of the increased blood viscosity and volume. • Treatment is directed toward reducing blood volume/viscosity and bone marrow activity.

Etiology Iron-deficiency anemia p609

may develop because of inadequate dietary intake, malabsorption, blood loss, or hemolysis. Normal dietary iron intake is usually enough to meet the needs of men and older women. It may be inadequate for people with higher iron needs (e.g., menstruating or pregnant women).

P631.20. The absolute neutrophil count (ANC) is determined by

multiplying the total WBC count by the percent of neutrophils. Neutropenia is defined as ANC less than 1000 cells/μL (1 × 109/L). Normally, neutrophils range from 2200 to 7700 cells/μL. Severe neutropenia is defined as an ANC less than 500 cells/μL.21>?????? Filgrastim increases the neutrophil count and function in neutropenic patients. Although total lymphocyte, platelet, and reticulocyte counts are also important to monitor in this patient, the absolute neutrophil count is used to evaluate the effects of filgrastim

A nurse is educating a patient who has recently been diagnosed with Hodgkin's lymphoma. The nurse explains which signs/symptoms may be experienced if the disease is in the advanced stage?

o Urinary symptoms Advanced stages of Hodgkin's lymphoma may be marked by enlarged retroperitoneal nodes, interfering with renal function and causing urinary symptoms. o Loss of motor function The advanced stages of Hodgkin's lymphoma may result in paraplegia and loss of motor function as a result of spinal cord compression. o Decreased red blood cell levels Anemia can result from increased destruction and decreased production of erythrocytes in the advanced stage of Hodgkin's lymphoma.

A nurse is caring for a patient with multiple myeloma who is experiencing significant skeletal pain. The patient expresses an interest in nonpharmacological pain relief. About which therapy would the nurse provide the patient more information?

o Use of a back brace Orthopedic support may help to reduce skeletal pain and does not require use of drugs.

hemophilia A

(classic hemophilia, factor VIII deficiency) Factor VIII is made in the liver and circulates as a complex with vWF. von Willebrand disease is a related disorder involving a deficiency of the von Willebrand coagulation protein . von Willebrand disease is the most common congenital bleeding disorder.

ACUTE MYELOGENOUS LEUKEMIA (AML)

- increase in the number of myeloblast . accounts for 15%-20% of acute leukemia in children and 80% in adults

heparin-Induced thrombocytopenia (HIT)

-HIT is a life-threatening condition and should be suspected if the platelet count falls by more than 50% from baseline or falls to below 100,000/µL. -Symptoms of bleeding are rare because the platelet count rarely drops below 20,000/µL. -Platelet destruction and vascular endothelial injury are major responses to an immune-mediated response to heparin. Platelet factor 4 (PF4) binds to heparin. This complex binds to the platelet surface leading to further platelet activation and release of more PF4, thus creating a positive feedback loop. -Antibodies are created against this complex and removed prematurely from circulation leading to thrombocytopenia and platelet-fibrin thrombi. Patients are at greater risk for thrombosis than bleeding. The major complication of HIT is venous thrombosis. Thrombotic events typically include deep vein thrombosis (DVT), pulmonary embolism, limb ischemia thrombosis, thrombotic stroke, and myocardial infarction (MI). · · Collaboration Pearl · Be aware that patients receiving heparin either subcutaneously (prophylactic dose) or intravenously (therapeutic dose) are both susceptible to heparin induced thrombocytopenia (HIT), so the nurse should monitor platelet count as available.

Thrombocytopenia Acquired-Immune Caused by:

-Immune or idiopathic thrombocytopenic purpura (ITP) -Neonatal alloimmune thrombocytopenia

11.p615. The patient with hemolytic anemia has the general manifestations of anemia and specific manifestations related to this type of anemia (Table 30.3).

. Jaundice is likely because the increased destruction of RBCs causes an elevation in bilirubin levels. The spleen and liver may enlarge because of their hyperactivity, which is related to macrophage phagocytosis of the defective RBCs.

· Macroangiopathic

: physical trauma (prosthetic heart valves, extracorporeal circulation)

Thrombocytopenia: Diagnostic Studies

A bone marrow biopsy may be done in order to analyze the marrow. The results may show normal or increased numbers of megakaryocytes (precursors of platelets). The biopsy is done to rule out leukemia, aplastic anemia, and other myeloproliferative disorders.

The nurse is caring for a patient newly diagnosed with Hodgkin's lymphoma. Which statement by the patient indicates to the nurse that further education on the disease's etiology is needed? A) "I've gotten a lot of viruses." B) "I work around a lot of toxins at work." C) "I once had an Epstein-Barr infection." D) "My aunt and grandmother had Hodgkin's too."

ANS A "I've gotten a lot of viruses." Possible risk factors for Hodgkin's lymphoma do not include viral infections. Viral infections are associated with causing multiple myeloma.

Documentation reveals a patient's lymph nodes are abnormal and indicative of Reed-Sternberg cells. When the nurse is reviewing the patient's medical history, which finding is likely? A) Mononucleosis infection 10 years ago B) Repeated radiation exposure C) Hip fracture from a fall and osteoporosis D) Presence of the Philadelphia chromosome

ANS A -Mononucleosis infection 10 years ago Reed-Sternberg cells are indicative of Hodgkin's lymphoma. Risk factors for Hodgkin's lymphoma include a history of Epstein-Barr virus infection, so a previous mononucleosis infection is a likely finding. -Repeated radiation exposure Radiation exposure is associated with multiple myeloma. Reed-Sternberg cells are indicative of Hodgkin's lymphoma. Past radiation exposure would not be a likely finding in a patient with Reed-Sternberg cells. - Hip fracture from a fall and osteoporosis Skeletal fractures and osteoporosis are associated with multiple myeloma. Reed-Sternberg cells are associated with Hodgkin's lymphoma. A previous hip fracture and osteoporosis would not be likely findings in a patient with Reed-Sternberg cells. -Presence of the Philadelphia chromosome The Philadelphia chromosome is associated with acute lymphocytic leukemia, while Reed-Sternberg cells are indicative of Hodgkin's lymphoma. The presence of the Philadelphia chromosome would not be a likely finding in a patient with Reed-Sternberg cells.

Which test result places a patient with leukemia at high risk for developing infection? A) Platelet count 150,000/µL B) Absolute neutrophil count (ANC) 500 cells/µL C) Total red blood cell (RBC) count 4.3 × 10 6/µL D) Mean corpuscular hemoglobin concentration (MCHC) 35%

ANS B Rationale Neutropenia indicates an increased risk of developing infection. Platelet count, total RBC count, and MCHC are in the normal range.

? PAGE607 - 24. A patient with a documented history of glucose-6-phosphate dehydrogenase deficiency has presented to the emergency department with signs and symptoms including pallor, jaundice, and malaise. Which of the nurses assessment questions relates most directly to this patients hematologic disorder? A) When did you last have a blood transfusion? B) What medications have taken recently? C) Have you been under significant stress lately? D)Have you suffered any recent injuries?

ANS B B Feedback: Exacerbations of glucose-6-phosphate dehydrogenase deficiency are nearly always precipitated by medications. Blood transfusions, stress, and injury are less common triggers.

6. A nursing student is caring for a patient with acute myeloid leukemia who is preparing to undergo induction therapy. In preparing a plan of care for this patient, the student should assign the highest priority to which nursing diagnoses? A) Activity Intolerance B) Risk for Infection C) Acute Confusion D) Risk for Spiritual Distress

ANS B Feedback: Induction therapy places the patient at risk for infection, thus this is the priority nursing diagnosis. During the time of induction therapy, the patient is very ill, with bacterial, fungal, and occasional viral infections; bleeding and severe mucositis, which causes diarrhea; and marked decline in the ability to maintain adequate nutrition. Supportive care consists of administering blood products and promptly treating infections. Immobility, confusion, and spiritual distress are possible, but infection is the patients most acute physiologic threat.

A patient presents to the emergency department with drug-induced thrombotic thrombocytopenic purpura (TTP). The patient is currently on aspirin and clopidogrel. The patient is complaining of a fever and severe discomfort in the left lower quadrant. The nurse should anticipate which treatment plan from the primary health care provider to care for this patient? A) Increase aspirin dose B) Discontinue clopidogrel C) Prepare patient for splenectomy D) Prepare patient for platelet transfusion

ANS B_ Discontinue clopidogrel This patient is experiencing a drug-induced TTP, so the nurse would expect the health care provider to remove the offending agent(s), including clopidogrel.

32. A young man with a diagnosis of hemophilia A has been brought to emergency department after suffering a workplace accident resulting in bleeding. Rapid assessment has revealed the source of the patients bleeding and established that his vital signs are stable. What should be the nurses next action? A) Position the patient in a prone position to minimize bleeding. B) Establish IV access for the administration of vitamin K. C) Prepare for the administration of factor VIII. D) Administer a normal saline bolus to increase circulatory volume.

ANS C Feedback: Injuries in patients with hemophilia necessitate prompt administration of clotting factors. Vitamin K is not a treatment modality and a prone position will not be appropriate for all types and locations of wounds. A normal saline bolus is not indicated.

37. A young adult patient has received the news that her treatment for Hodgkin lymphoma has been deemed successful and that no further treatment is necessary at this time. The care team should ensure that the patient receives regular health assessments in the future due to the risk of what complication? A) Iron-deficiency anemia B) Hemophilia C) Hematologic cancers D) Genitourinary cancers

ANS C Feedback: Survivors of Hodgkin lymphoma have a high risk of second cancers, with hematologic cancers being the most common. There is no consequent risk of anemia or hemophilia, and hematologic cancers are much more common than GU cancers.

A patient with Hodgkin's lymphoma has been receiving chemotherapy for several months. Aside from managing chemotherapy side effects, which is the most appropriate intervention to be included in the patient's plan of care? A-Recommend appropriate rehabilitation resources once chemotherapy ends B-Suggest alternative therapies to replace the chemotherapy C-Support the patient by providing resources for optimal psychosocial health D-Have the patient adhere to a strict dietary pattern to ensure adequate caloric intake

ANS C Support the patient by providing resources for optimal psychosocial health Since survival of patients with Hodgkin's lymphoma depends on their response to treatment, one intervention to be included in the patient's plan of care is providing support for the patient by addressing the physical, psychological, social, and spiritual consequences of the patient's disease and its treatment.

38. The clinical nurse educator is presenting health promotion education to a patient who will be treated for non-Hodgkin lymphoma on an outpatient basis. The nurse should recommend which of the following actions? A) Avoiding direct sun exposure in excess of 15 minutes daily B) Avoiding grapefruit juice and fresh grapefruit C) Avoiding highly crowded public places D) Using an electric shaver rather than a razor

ANS C C Feedback: The risk of infection is significant for these patients, not only from treatment-related myelosuppression but also from the defective immune response that results from the disease itself. Limiting infection exposure is thus necessary. The need to avoid grapefruit is dependent on the patients medication regimen. Sun exposure and the use of razors are not necessarily contraindicated.

23. A nurse is caring for a patient who is being treated for leukemia in the hospital. The patient was able to maintain her nutritional status for the first few weeks following her diagnosis but is now exhibiting early signs and symptoms of malnutrition. In collaboration with the dietitian, the nurse should implement what Intervention? A) Arrange for total parenteral nutrition (TPN). B) Facilitate placement of a percutaneous endoscopic gastrostomy (PEG) tube. C) Provide the patient with several small, soft-textured meals each day. D) Assign responsibility for the patients nutrition to the patients friends and family.

ANS C FEEDBACK For patients experiencing difficulties with oral intake, the provision of small, easily chewed meals may be beneficial. This option would be trialed before resorting to tube feeding or TPN. The family should be encouraged to participate in care, but should not be assigned full responsibility.

In which patient with thrombocytopenia would a platelet transfusion be appropriate? A) A patient with heparin-induced therapy (HIT) and platelet count <10,000/μL B) A patient with thrombocytopenic purpura (ITP), painful splenomegaly, and platelet count <60,000/μL C) A patient with heparin-induced therapy (HIT) and a platelet count <30,000/μL D) A patient with immune thrombocytopenic purpura (ITP) with a platelet count <10, 000/μL

ANS D A patient with immune thrombocytopenic purpura (ITP) with a platelet count <10, 000/μL A patient with ITP and a platelet count <10,000 will receive a platelet transfusion. Usually ITP can be treated effectively with steroids, but if this treatment fails and the platelet level falls, the patient will need a transfusion.

23. The medical nurse is aware that patients with sickle cell anemia benefit from understanding what situations can precipitate a sickle cell crisis. When teaching a patient with sickle cell anemia about strategies to prevent crises, what measures should the nurse recommend? A) Using prophylactic antibiotics and performing meticulous hygiene B) Maximizing physical activity and taking OTC iron supplements C) Limiting psychosocial stress and eating a high-protein diet D) Avoiding cold temperatures and ensuring sufficient hydration

ANS D Feedback: Keeping warm and providing adequate hydration can be effective in diminishing the occurrence and severity of attacks. Hygiene, antibiotics, and high protein intake do not prevent crises. Maximizing activity may exacerbate pain and be unrealistic.

Iron-Deficiency Anemia p609

the most common nutritional disorder in the world. Those most susceptible to iron-deficiency anemia are the very young, those on poor diets, and women in their reproductive years. Normally, we lose 1 mg of iron daily in urine, bile, sweat, sloughing of epithelial cells from the skin and intestinal mucosa, and minor bleeding.

9. A nurse is caring for a patient with severe anemia. The patient is tachycardic and complains of dizziness and exertional dyspnea. The nurse knows that in an effort to deliver more blood to hypoxic tissue, the workload on the heart is increased. What signs and symptoms might develop if this patient goes into heart failure? A) Peripheral edema B)Nausea and vomiting C) Migraine D) Fever

ANS: A Feedback: Cardiac status should be carefully assessed in patients with anemia. When the hemoglobin level is low, the heart attempts to compensate by pumping faster and harder in an effort to deliver more blood to hypoxic tissue. This increased cardiac workload can result in such symptoms as tachycardia, palpitations, dyspnea, dizziness, orthopnea, and exertional dyspnea. Heart failure may eventually develop, as evidenced by an enlarged heart (cardiomegaly) and liver (hepatomegaly), and by peripheral edema. Nausea, migraine, and fever are not associated with heart failure.

28. A nurse at a blood donation clinic has completed the collection of blood from a woman. The woman states that she feels lightheaded and she appears visibly pale. What is the nurses most appropriate action? A) Help her into a sitting position with her head lowered below her knees. B) Administer supplementary oxygen by nasal prongs. C) Obtain a full set of vital signs. D) Inform a physician or other primary care provider.

ANS: A Feedback: A donor who appears pale or complains of faintness should immediately lie down or sit with the head lowered below the knees. He or she should be observed for another 30 minutes. There is no immediate need for a physicians care. Supplementary oxygen may be beneficial, but may take too much time to facilitate before a syncopal episode. Repositioning must precede assessment of vital signs.

The nurse is educating a patient on the appropriate treatment for Hodgkin's lymphoma. Which statement by the patient indicates the need for further teaching? Select all that apply. A)"I will be able to stop my treatment once the cancer is in remission." B)"We can start with a conservative treatment of corticosteroids and see how I respond." C) "The treatment for the disease is safe and without serious side effects." D) "The treatment for my cancer will depend on the stage at which I am diagnosed." E) "Chemotherapy is the only treatment for Hodgkin's lymphoma."

ANS: A,B,C,E I will be able to stop my treatment once the cancer is in remission." Once in remission, intensive chemotherapy may be needed to achieve a cure. Therapy should not be discontinued. "We can start with a conservative treatment of corticosteroids and see how I respond." Corticosteroid treatment is the initial treatment for multiple myeloma. Hodgkin's lymphoma is treated more aggressively at the outset. "The treatment for the disease is safe and without serious side effects." The treatment for Hodgkin's lymphoma is aggressive and may result in life-threatening side effects. "The treatment for my cancer will depend on the stage at which I am diagnosed." The treatment for Hodgkin's lymphoma depends on the stage and prognosis. "Chemotherapy is the only treatment for Hodgkin's lymphoma." Irradiation may be used as a supplement to chemotherapy, depending on site of disease and the presence of resistant disease after chemotherapy.

45. Which information obtained by the nurse assessing a patient admitted with multiple myeloma is most important to report to the health care provider? A) Patient reports severe back pain. B) Serum calcium level is 15 mg/dL. C) Patient reports no stool for 5 days. D) Urine sample has Bence-Jones protein.

ANS: B Hypercalcemia may lead to complications such as dysrhythmias or seizures and should be addressed quickly. The other patient findings will also be discussed with the health care provider but are not life threatening.

Thrombocytosis is defined as

too many platelets. It occurs with inflammation and some cancers (see Chapter 30). The most likely complication of thrombocytosis is excessive clotting.

In severe anemia (Hgb <6 g/dL [<60 g/L])

· Pallor or jaundice · Pruritus · Icteric conjunctiva and sclera · Retinal hemorrhage · Blurred vision · Glossitis or a smooth tongue · Tachycardia · Increased pulse pressure · Systolic murmurs · Intermittent claudication · Angina · Heart failure or myocardial infarction (MI) · Tachypnea, orthopnea, dyspnea at rest · Headache or vertigo · Irritability, depression, impaired thought processes · Anorexia · Hepatomegaly, splenomegaly

Hodgkin's Lymphoma

· Peripheral blood analysis · Excisional lymph node biopsy · Bone marrow examination · Radiological studies Positron emission tomography (PET) scans with or without computed tomography (CT) scan are used to stage and then assess response to therapy.

A patient is diagnosed with thrombocytopenia. Which symptoms would the nurse expect to observe in a patient with thrombocytopenia?

· Tachycardia · Tachycardia is a clinical manifestation of thrombocytopenia that can occur due to internal bleeding. · Hypotension · Hypotension is a clinical manifestation of thrombocytopenia that can occur due to internal bleeding. · Abdominal pain · Abdominal pain can be a symptom of internal bleeding and a clinical manifestation of thrombocytopenia

33. A nurse is planning the care of a patient who has a diagnosis of hemophilia A. When addressing the nursing diagnosis of Acute Pain Related to Joint Hemorrhage, what principle should guide the nurses choice of interventions? A) Gabapentin (Neurontin) is effective because of the neuropathic nature of the patients pain. B) Opioids partially inhibit the patients synthesis of clotting factors. C) Opioids may cause vasodilation and exacerbate bleeding. D) NSAIDs are contraindicated due to the risk for bleeding.

ANS: D Feedback: NSAIDs may be contraindicated in patients with hemophilia due to the associated risk of bleeding. Opioids do not have a similar effect and they do not inhibit platelet synthesis. The pain associated with hemophilia is not neuropathic.

Myelodysplastic Syndrome

• Myelodysplastic syndrome (MDS) is a group of related hematologic disorders characterized by a change in the quantity and quality of bone marrow elements. Although it can occur in all age groups, the highest prevalence is in people over 80 years of age. • Supportive treatment consists of hematologic monitoring, antibiotic therapy, or transfusions with blood products along with iron chelators to prevent iron overload. • The overall goal is to improve hematopoiesis and ensure age-related quality of life.

12. A patient has been diagnosed with a lymphoid stem cell defect. This patient has the potential for a problem involving which of the following? A) Plasma cells B) Neutrophils C) Red blood cells D) Platelets

Ans: A Feedback: A defect in a myeloid stem cell can cause problems with erythrocyte, leukocyte, and platelet production. In contrast, a defect in the lymphoid stem cell can cause problems with T or B lymphocytes, plasma cells (a more differentiated form of B lymphocyte), or natural killer (NK) cells.

7. A patient comes to the clinic complaining of fatigue and the health interview is suggestive of pica. Laboratory findings reveal a low serum iron level and a low ferritin level. With what would the nurse suspect that the patient will be diagnosed? A) Iron deficiency anemia B) Pernicious anemia C) Sickle cell anemia D) Hemolytic anemia

Ans: A Feedback: A low serum iron level, a low ferritin level, and symptoms of pica are associated with iron deficiency anemia. TIBC may also be elevated. None of the other anemias are associated with pica.

19. A patients most recent blood work reveals low levels of albumin. This assessment finding should suggest the possibility of what nursing diagnosis? A) Risk for imbalanced fluid volume related to low albumin B) Risk for infection related to low albumin C) Ineffective tissue perfusion related to low albumin D) Impaired skin integrity related to low albumin

Ans: A Feedback: Albumin is particularly important for the maintenance of fluid balance within the vascular system. Deficiencies nearly always manifest as fluid imbalances. Tissue oxygenation and skin integrity are not normally affected. Low albumin does not constitute a risk for infection.

30. A patient with a recent diagnosis of ITP has asked the nurse why the care team has not chosen to administer platelets, stating, I have low platelets, so why not give me a transfusion of exactly what Im missing? How should the nurse best respond? A) Transfused platelets usually arent beneficial because theyre rapidly destroyed in the body. B) A platelet transfusion often blunts your bodys own production of platelets even further. C) Finding a matching donor for a platelet transfusion is exceedingly difficult. D) A very small percentage of the platelets in a transfusion are actually functional.

Ans: A Feedback: Despite extremely low platelet counts, platelet transfusions are usually avoided. Transfusions tend to be ineffective not because the platelets are nonfunctional but because the patients antiplatelet antibodies bind with the transfused platelets, causing them to be destroyed. Matching the patients blood type is not usually necessary for a platelet transfusion. Platelet transfusions do not exacerbate low platelet production.

3. An oncology nurse is caring for a patient with multiple myeloma who is experiencing bone destruction. When reviewing the patients most recent blood tests, the nurse should anticipate what imbalance? A) Hypercalcemia B) Hyperproteinemia C) Elevated serum viscosity D) Elevated RBC count

Ans: A Feedback: Hypercalcemia may result when bone destruction occurs due to the disease process. Elevated serum viscosity occurs because plasma cells excrete excess immunoglobulin. RBC count will be decreased. Hyperproteinemia would not be present.

2. A nurse is caring for a patient who has a diagnosis of acute leukemia. What assessment most directly addresses the most common cause of death among patients with leukemia? A) Monitoring for infection B) Monitoring nutritional status C) Monitor electrolyte levels D) Monitoring liver function

Ans: A Feedback: In patients with acute leukemia, death typically occurs from infection or bleeding. Compromised nutrition, electrolyte imbalances, and impaired liver function are all plausible, but none is among the most common causes of death in this patient population.

22. A patient has been found to have an indolent neoplasm. The nurse should recognize what implication of this condition? A) The patient faces a significant risk of malignancy. B) The patient has a myeloid form of leukemia. C) The patient has a lymphocytic form of leukemia. D) The patient has a major risk factor for hemophilia.

Ans: A Feedback: Indolent neoplasms have the potential to develop into a neoplasm, but this is not always the case. The patient does not necessary have, or go on to develop, leukemia. Indolent neoplasms are unrelated to the pathophysiology of hemophilia.

18. A patient with a diagnosis of acute myeloid leukemia (AML) is being treated with induction therapy on the oncology unit. What nursing action should be prioritized in the patients care plan? A) Protective isolation and vigilant use of standard precautions B) Provision of a high-calorie, low-texture diet and appropriate oral hygiene C) Including the family in planning the patients activities of daily living D) Monitoring and treating the patients pain

Ans: A Feedback: Induction therapy causes neutropenia and a severe risk of infection. This risk must be addressed directly in order to ensure the patients survival. For this reason, infection control would be prioritized over nutritional interventions, family care, and pain, even though each of these are important aspects of nursing care.

15. An older adult client is exhibiting many of the characteristic signs and symptoms of iron deficiency. In addition to a complete blood count, what diagnostic assessment should the nurse anticipate? A) Stool for occult blood B) Bone marrow biopsy C) Lumbar puncture D) Urinalysis

Ans: A Feedback: Iron deficiency in the adult generally indicates blood loss (e.g., from bleeding in the GI tract or heavy menstrual flow). Bleeding in the GI tract can be preliminarily identified by testing stool for the presence of blood. A bone marrow biopsy would not be undertaken for the sole purpose of investigating an iron deficiency. Lumbar puncture and urinalysis would not be clinically relevant.

1. An oncology nurse is providing health education for a patient who has recently been diagnosed with leukemia. What should the nurse explain about commonalities between all of the different subtypes of leukemia? A) The different leukemias all involve unregulated proliferation of white blood cells. B) The different leukemias all have unregulated proliferation of red blood cells and decreased bone marrow function. C) The different leukemias all result in a decrease in the production of white blood cells. D) The different leukemias all involve the development of cancer in the lymphatic system.

Ans: A Feedback: Leukemia commonly involves unregulated proliferation of white blood cells. Decreased production of red blood cells is associated with anemias. Decreased production of white blood cells is associated with leukopenia. The leukemias are not characterized by their involvement with the lymphatic system.

20. A 60-year-old patient with chronic myeloid leukemia will be treated in the home setting and the nurse is preparing appropriate health education. What topic should the nurse emphasize? A) The importance of adhering to the prescribed drug regimen B) The need to ensure that vaccinations are up to date C) The importance of daily physical activity D) The need to avoid shellfish and raw foods

Ans: A Feedback: Nurses need to understand that the effectiveness of the drugs used to treat CML is based on the ability of the patient to adhere to the medication regimen as prescribed. Adherence is often incomplete, thus this must be a focus of health education. Vaccinations normally would not be administered during treatment and daily physical activity may be impossible for the patient. Dietary restrictions are not normally necessary.

33. A nurse is planning the care of a patient who has been diagnosed with essential thrombocythemia (ET). What nursing diagnosis should the nurse prioritize when choosing interventions? A) Risk for Ineffective Tissue Perfusion B) Risk for Imbalanced Fluid Volume C) Risk for Ineffective Breathing Pattern D) Risk for Ineffective Thermoregulation

Ans: A Feedback: Patients with ET are at risk for hypercoagulation and consequent ineffective tissue perfusion. Fluid volume, breathing, and thermoregulation are not normally affected.

14. A patient with a history of cirrhosis is admitted to the ICU with a diagnosis of bleeding esophageal varices; an attempt to stop the bleeding has been only partially successful. What would the critical care nurse expect the care team to order for this patient? A) Packed red blood cells (PRBCs) B) Vitamin K C) Oral anticoagulants D) Heparin infusion

Ans: A Feedback: Patients with liver dysfunction may have life-threatening hemorrhage from peptic ulcers or esophageal varices. In these cases, replacement with fresh frozen plasma, PRBCs, and platelets is usually required. Vitamin K may be ordered once the bleeding is stopped, but that is not what is needed to stop the bleeding of the varices. Anticoagulants would exacerbate the patients bleeding.

26. The nurses brief review of a patients electronic health record indicates that the patient regularly undergoes therapeutic phlebotomy. Which of the following rationales for this procedure is most plausible? A) The patient may chronically produce excess red blood cells. B) The patient may frequently experience a low relative plasma volume. C) The patient may have impaired stem cell function. D) The patient may previously have undergone bone marrow biopsy.

Ans: A Feedback: Persistently elevated hematocrit is an indication for therapeutic phlebotomy. It is not used to address excess or deficient plasma volume and is not related to stem cell function. Bone marrow biopsy is not an indication for therapeutic phlebotomy.

26. A patients absolute neutrophil count (ANC) is 440/mm3. But the nurses assessment reveals no apparent signs or symptoms of infection. What action should the nurse prioritize when providing care for this patient? A) Meticulous hand hygiene B) Timely administration of antibiotics C) Provision of a nutrient-dense diet D) Maintaining a sterile care environment

Ans: A Feedback: Providing care for a patient with neutropenia requires that the nurse adhere closely to standard precautions and infection control procedures. Hand hygiene is central to such efforts. Prophylactic antibiotics are rarely used and it is not possible to provide a sterile environment for care. Nutrition is highly beneficial, but hand hygiene is the central aspect of care.

28. Following an extensive diagnostic workup, an older adult patient has been diagnosed with a secondary myelodysplastic syndrome (MDS). What assessment question most directly addresses the potential etiology of this patients health problem? A) Were you ever exposed to toxic chemicals in any of the jobs that you held? B) When you were younger, did you tend to have recurrent infections of any kind? C) Have your parents or siblings had any disease like this? D) Would you say that youve had a lot of sun exposure in your lifetime?

Ans: A Feedback: Secondary MDS can occur at any age and results from prior toxic exposure to chemicals, including chemotherapeutic medications. Family history, sun exposure, and previous infections are unrelated to the pathophysiology of secondary MDS.

13. A nurse is caring for a patient with Hodgkin lymphoma at the oncology clinic. The nurse should be aware of what main goal of care? A) Cure of the disease B) Enhancing quality of life C) Controlling symptoms D) Palliation

Ans: A Feedback: The goal in the treatment of Hodgkin lymphoma is cure. Palliation is thus not normally necessary. Quality of life and symptom control are vital, but the overarching goal is the cure the disease.

38. The nurse is preparing to administer a unit of platelets to an adult patient. When administering this blood product, which of the following actions should the nurse perform? A) Administer the platelets as rapidly as the patient can tolerate. B) Establish IV access as soon as the platelets arrive from the blood bank. C) Ensure that the patient has a patent central venous catheter. D) Aspirate 10 to 15 mL of blood from the patients IV immediately following the transfusion.

Ans: A Feedback: The nurse should infuse each unit of platelets as fast as patient can tolerate to diminish platelet clumping during administration. IV access should be established prior to obtaining the platelets from the blood bank. A central line is appropriate for administration, but peripheral IV access (22-gauge or larger) is sufficient. There is no need to aspirate after the transfusion.

34. A patient is receiving a blood transfusion and complains of a new onset of slight dyspnea. The nurses rapid assessment reveals bilateral lung crackles and elevated BP. What is the nurses most appropriate action? A) Slow the infusion rate and monitor the patient closely. B) Discontinue the transfusion and begin resuscitation. C) Pause the transfusion and administer a 250 mL bolus of normal saline. D) Discontinue the transfusion and administer a beta-blocker, as ordered.

Ans: A Feedback: The patient is showing early signs of hypervolemia; the nurse should slow the infusion rate and assess the patient closely for any signs of exacerbation. At this stage, discontinuing the transfusion is not necessary. A bolus would worsen the patients fluid overload.

9. A patient with leukemia has developed stomatitis and is experiencing a nutritional deficit. An oral anesthetic has consequently been prescribed. What health education should the nurse provide to the patient? A) Chew with care to avoid inadvertently biting the tongue. B) Use the oral anesthetic 1 hour prior to meal time. C) Brush teeth before and after eating. D) Swallow slowly and deliberately.

Ans: A If oral anesthetics are used, the patient must be warned to chew with extreme care to avoid inadvertently biting the tongue or buccal mucosa. An oral anesthetic would be metabolized by the time the patient eats if it is used 1 hour prior to meals. There is no specific need to warn the patient about brushing teeth or swallowing slowly because an oral anesthetic has been used.

35. A nurse is writing the care plan of a patient who has been diagnosed with myelofibrosis. What nursing diagnoses should the nurse address? Select all that apply. A) Disturbed Body Image B) Impaired Mobility C) Imbalanced Nutrition: Less than Body Requirements D) Acute Confusion E) Risk for Infection

Ans: A, B, C, E Feedback: The profound splenomegaly that accompanies myelofibrosis can impact the patients body image and mobility. As well, nutritional deficits are common and the patient is at risk for infection. Cognitive effects are less common.

29. The results of a patients most recent blood work and physical assessment are suggestive of immune thrombocytopenic purpura (ITP). This patient should undergo testing for which of the following potential causes? Select all that apply. A) Hepatitis B) Acute renal failure C) HIV D) Malignant melanoma E) Cholecystitis

Ans: A, C Feedback: Viral illnesses have the potential to cause ITP. Renal failure, malignancies, and gall bladder inflammation are not typical causes of ITP.

25. An oncology nurse recognizes a patients risk for fluid imbalance while the patient is undergoing treatment for leukemia. What relevant assessments should the nurse include in the patients plan of care? Select all that apply. A) Monitoring the patients electrolyte levels B) Monitoring the patients hepatic function C) Measuring the patients weight on a daily basis D) Measuring and recording the patients intake and output E) Auscultating the patients lungs frequently

Ans: A, C, D, E Feedback: Assessments that relate to fluid balance include monitoring the patients electrolytes, auscultating the patients chest for adventitious sounds, weighing the patient daily, and closely monitoring intake and output. Liver function is not directly relevant to the patients fluid status in most cases.

9. Through the process of hematopoiesis, stem cells differentiate into either myeloid or lymphoid stem cells. Into what do myeloid stem cells further differentiate? Select all that apply. A) Leukocytes B) Natural killer cells C) Cytokines D) Platelets E) Erythrocytes

Ans: A, D, E Feedback: Myeloid stem cells differentiate into three broad cell types: erythrocytes, leukocytes, and platelets. Natural killer cells and cytokines do not originate as myeloid stem cells.

32. A patient is being treated in the ICU after a medical error resulted in an acute hemolytic transfusion reaction. What was the etiology of this patients adverse reaction? A) Antibodies to donor leukocytes remained in the blood. B) The donor blood was incompatible with that of the patient. C) The patient had a sensitivity reaction to a plasma protein in the blood. D) The blood was infused too quickly and overwhelmed the patients circulatory system.

Ans: B An acute hemolytic reaction occurs when the donor blood is incompatible with that of the recipient. In the case of a febrile nonhemolytic reaction, antibodies to donor leukocytes remain in the unit of blood or blood component. An allergic reaction is a sensitivity reaction to a plasma protein within the blood component. Hypervolemia does not cause an acute hemolytic reaction.

16. A nurse practitioner is assessing a patient who has a fever, malaise, and a white blood cell count that is elevated. Which of the following principles should guide the nurses management of the patients care? A) There is a need for the patient to be assessed for lymphoma. B) Infection is the most likely cause of the patients change in health status. C) The patient is exhibiting signs and symptoms of leukemia. D) The patient should undergo diagnostic testing for multiple myeloma.

Ans: B Feedback: Leukocytosis is most often the result of infection. It is only considered pathologic (and suggestive of leukemia) if it is persistent and extreme. Multiple myeloma and lymphoma are not likely causes of this constellation of symptoms.

40. A patients electronic health record states that the patient receives regular transfusions of factor IX. The nurse would be justified in suspecting that this patient has what diagnosis? A) Leukemia B) Hemophilia C) Hypoproliferative anemia D) Hodgkins lymphoma

Ans: B Feedback: Administration of clotting factors is used to treat diseases where these factors are absent or insufficient; hemophilia is among the most common of these diseases. Factor IX is not used in the treatment of leukemia, lymphoma, or anemia.

25. A patient is scheduled for a splenectomy. During discharge education, what teaching point should the nurse prioritize? A) The importance of adhering to prescribed immunosuppressant therapy B) The need to report any signs or symptoms of infection promptly C) The need to ensure adequate folic acid, iron, and vitamin B12 intake D) The importance of limiting activity postoperatively to prevent hemorrhage

Ans: B Feedback: After splenectomy, the patient is instructed to seek prompt medical attention if even relatively minor symptoms of infection occur. Often, patients with high platelet counts have even higher counts after splenectomy, which can predispose them to serious thrombotic or hemorrhagic problems. However, this increase is usually transient and therefore often does not warrant additional treatment. modifications are not normally necessary and immunosuppressants would be strongly contraindicated.

21. An older adult patient is undergoing diagnostic testing for chronic lymphocytic leukemia (CLL). What assessment finding is certain to be present if the patient has CLL? A) Increased numbers of blast cells B) Increased lymphocyte levels C) Intractable bone pain D) Thrombocytopenia with no evidence of bleeding

Ans: B Feedback: An increased lymphocyte count (lymphocytosis) is always present in patients with CLL. Each of the other listed symptoms may or may not be present, and none is definitive for CLL.

25. A patients electronic health record notes that he has previously undergone treatment for secondary polycythemia. How should this aspect of the patients history guide the nurses subsequent assessment? A)The nurse should assess for recent blood donation. B) The nurse should assess for evidence of lung disease. C) The nurse should assess for a history of venous thromboembolism. D) The nurse should assess the patient for impaired renal function.

Ans: B Feedback: Any reduction in oxygenation, such as lung disease, can cause secondary polycythemia. Blood donation does not precipitate this problem and impaired renal function typically causes anemia, not polycythemia. A history of VTE is not a likely contributor.

30. A nurse is preparing health education for a patient who has received a diagnosis of myelodysplastic syndrome (MDS). Which of the following topics should the nurse prioritize? A) Techniques for energy conservation and activity management B) Emergency management of bleeding episodes C) Technique for the administration of bronchodilators by metered-dose inhaler D) Techniques for self-palpation of the lymph nodes

Ans: B Feedback: Because of patients risks of hemorrhage, patients with MDS should be taught techniques for managing emergent bleeding episodes. Bronchodilators are not indicated for the treatment of MDS and lymphedema is not normally associated with the disease. Energy conservation techniques are likely to be useful, but management of hemorrhage is a priority because of the potential consequences.

30. A patient on the medical unit is receiving a unit of PRBCs. Difficult IV access has necessitated a slow infusion rate and the nurse notes that the infusion began 4 hours ago. What is the nurses most appropriate action? A) Apply an icepack to the blood that remains to be infused. B) Discontinue the remainder of the PRBC transfusion and inform the physician. C) Disconnect the bag of PRBCs, cool for 30 minutes and then administer. D) Administer the remaining PRBCs by the IV direct (IV push) route.

Ans: B Feedback: Because of the risk of infection, a PRBC transfusion should not exceed 4 hours. Remaining blood should not be transfused, even if it is cooled. Blood is not administered by the IV direct route.

16. A patient is being treated for the effects of a longstanding vitamin B12 deficiency. What aspect of the patients health history would most likely predispose her to this deficiency? A) The patient has irregular menstrual periods. B) The patient is a vegan. C) The patient donated blood 60 days ago. D) The patient frequently smokes marijuana.

Ans: B Feedback: Because vitamin B12 is found only in foods of animal origin, strict vegetarians may ingest little vitamin B12. Irregular menstrual periods, marijuana use, and blood donation would not precipitate a vitamin B12 deficiency.

21. The nurse is providing care for an older adult who has a hematologic disorder. What age-related change in hematologic function should the nurse integrate into care planning? A) Bone marrow in older adults produces a smaller proportion of healthy, functional blood cells. B) Older adults are less able to increase blood cell production when demand suddenly increases. C) Stem cells in older adults eventually lose their ability to differentiate. D) The ratio of plasma to erythrocytes and lymphocytes increases with age.

Ans: B Feedback: Due to a variety of factors, when an older person needs more blood cells, the bone marrow may not be able to increase production of these cells adequately. Stem cell activity continues throughout the lifespan, although at a somewhat decreased rate. The proportion of functional cells does not greatly decrease and the relative volume of plasma does not change significantly.

20. An individual has accidentally cut his hand, immediately initiating the process of hemostasis. Following vasoconstriction, what event in the process of hemostasis will take place? A) Fibrin will be activated at the bleeding site. B) Platelets will aggregate at the injury site. C) Thromboplastin will form a clot. D) Prothrombin will be converted to thrombin.

Ans: B Feedback: Following vasoconstriction, circulating platelets aggregate at the site and adhere to the vessel and to one another, forming an unstable hemostatic plug. Events involved in the clotting cascade take place subsequent to this initial platelet action.

27. A nurse has participated in organizing a blood donation drive at a local community center. Which of the following individuals would most likely be disallowed from donating blood? A) A man who is 81 years of age B) A woman whose blood pressure is 88/51 mm Hg C) A man who donated blood 4 months ago D) A woman who has type 1 diabetes

Ans: B Feedback: For potential blood donors, systolic arterial BP should be 90 to 180 mm Hg, and the diastolic pressure should be 50 to 100 mm Hg. There is no absolute upper age limit. Donation 4 months ago does not preclude safe repeat donation and diabetes is not a contraindication.

15. An adult patient has presented to the health clinic with a complaint of a firm, painless cervical lymph node. The patient denies any recent infectious diseases. What is the nurses most appropriate response to the patients complaint? A) Call 911. B) Promptly refer the patient for medical assessment. C) Facilitate a radiograph of the patients neck and have the results forwarded to the patients primary care provider. D) Encourage the patient to track the size of the lymph node and seek care in 1 week.

Ans: B Feedback: Hodgkin lymphoma usually begins as an enlargement of one or more lymph nodes on one side of the neck. The individual nodes are painless and firm but not hard. Prompt medical assessment is necessary if a patient has this presentation. However, there is no acute need to call 911. Delaying care for 1 week could have serious consequences and x-rays are not among the common diagnostic tests.

34. A night nurse is reviewing the next days medication administration record (MAR) of a patient who has hemophilia. The nurse notes that the MAR specifies both oral and subcutaneous options for the administration of a PRN antiemetic. What is the nurses best action? A) Ensure that the day nurse knows not to give the antiemetic. B) Contact the prescriber to have the subcutaneous option discontinued. C) Reassess the patients need for antiemetics. D) Remove the subcutaneous route from the patients MAR.

Ans: B Feedback: Injections must be avoided in patients with hemophilia. Consequently, the nurse should ensure that the prescriber makes the necessary change. The nurse cannot independently make a change to a patients MAR in most cases. Facilitating the necessary change is preferable to deferring to the day nurse.

13. The nurse is describing normal RBC physiology to a patient who has a diagnosis of anemia. The nurse should explain that the RBCs consist primarily of which of the following? A) Plasminogen B) Hemoglobin C) Hematocrit D) Fibrin

Ans: B Feedback: Mature erythrocytes consist primarily of hemoglobin, which contains iron and makes up 95% of the cell mass. RBCs are not made of fibrin or plasminogen. Hematocrit is a measure of RBC volume in whole blood.

10. A patient diagnosed with acute myelogenous leukemia has just been admitted to the oncology unit. When writing this patients care plan, what potential complication should the nurse address? A) Pancreatitis B) Hemorrhage C) Arteritis D) Liver dysfunction

Ans: B Feedback: Pancreatitis, arteritis, and liver dysfunction are generally not complications of leukemia. However, the patient faces a high risk of hemorrhage

27. A nurse is providing discharge education to a patient who has recently been diagnosed with a bleeding disorder. What topic should the nurse prioritize when teaching this patient? A) Avoiding buses, subways, and other crowded, public sites B) Avoiding activities that carry a risk for injury C) Keeping immunizations current D) Avoiding foods high in vitamin K

Ans: B Feedback: Patients with bleeding disorders need to understand the importance of avoiding activities that increase the risk of bleeding, such as contact sports. Immunizations involve injections and may be contraindicated for some patients. Patients with bleeding disorders do not need to normally avoid crowds. Foods high in vitamin K may beneficial, not detrimental.

10. A patient is admitted to the hospital with pernicious anemia. The nurse should prepare to administer which of the following medications? A) Folic acid B) Vitamin B12 C) Lactulose D) Magnesium sulfate

Ans: B Feedback: Pernicious anemia is characterized by vitamin B12 deficiency. Magnesium sulfate, lactulose, and folic acid do not address the pathology of this type of anemia.

39. A patient has a diagnosis of multiple myeloma and the nurse is preparing health education in preparation for discharge from the hospital. What action should the nurse promote? A) Daily performance of weight-bearing exercise to prevent muscle atrophy B) Close monitoring of urine output and kidney function C) Daily administration of warfarin (Coumadin) as ordered D) Safe use of supplementary oxygen in the home setting

Ans: B Feedback: Renal function must be monitored closely in the patient with multiple myeloma. Excessive weight- bearing can cause pathologic fractures. There is no direct indication for anticoagulation or supplementary oxygen.

24. A patient who is undergoing consolidation therapy for the treatment of leukemia has been experiencing debilitating fatigue. How can the nurse best meet this patients needs for physical activity? A) Teach the patient about the risks of immobility and the benefits of exercise. B) Assist the patient to a chair during awake times, as tolerated. C) Collaborate with the physical therapist to arrange for stair exercises. D) Teach the patient to perform deep breathing and coughing exercises.

Ans: B Feedback: Sitting is a chair is preferable to bed rest, even if a patient is experiencing severe fatigue. A patient who has debilitating fatigue would not likely be able to perform stair exercises. Teaching about mobility may be necessary, but education must be followed by interventions that actually involve mobility. Deep breathing and coughing reduce the risk of respiratory complications but are not substitutes for physical mobility in preventing deconditioning.

36. A patient is receiving the first of two ordered units of PRBCs. Shortly after the initiation of the transfusion, the patient complains of chills and experiences a sharp increase in temperature. What is the nurses priority action? A) Position the patient in high Fowlers. B) Discontinue the transfusion. C) Auscultate the patients lungs. D) Obtain a blood specimen from the patient.

Ans: B Feedback: Stopping the transfusion is the first step in any suspected transfusion reaction. This must precede other assessments and interventions, including repositioning, chest auscultation, and collecting specimens.

6. A patient with renal failure has decreased erythropoietin production. Upon analysis of the patients complete blood count, the nurse will expect which of the following results? A) An increased hemoglobin and decreased hematocrit B) A decreased hemoglobin and hematocrit C) A decreased mean corpuscular volume (MCV) and red cell distribution width (RDW) D) An increased MCV and RDW

Ans: B Feedback: The decreased production of erythropoietin will result in a decreased hemoglobin and hematocrit. The patient will have normal MCV and RDW because the erythrocytes are normal in appearance.

16. A group of nurses are learning about the high incidence and prevalence of anemia among different populations. Which of the following individuals is most likely to have anemia? A) A 50-year-old African-American woman who is going through menopause B) An 81-year-old woman who has chronic heart failure C) A 48-year-old man who travels extensively and has a high-stress job D) A 13-year-old girl who has just experienced menarche

Ans: B Feedback: The incidence and prevalence of anemia are exceptionally high among older adults, and the risk of anemia is compounded by the presence of heart disease. None of the other listed individuals exhibits high-risk factors for anemia, though exceptionally heavy menstrual flow can result in anemia.

36. An adult patients abnormal complete blood count (CBC) and physical assessment have prompted the primary care provider to order a diagnostic workup for Hodgkin lymphoma. The presence of what assessment finding is considered diagnostic of the disease? A) Schwann cells B) Reed-Sternberg cells C) Lewy bodies D) Loops of Henle

Ans: B Feedback: The malignant cell of Hodgkin lymphoma is the Reed-Sternberg cell, a gigantic tumor cell that is morphologically unique and thought to be of immature lymphoid origin. It is the pathologic hallmark and essential diagnostic criterion. Schwann cells exist in the peripheral nervous system and Lewy bodies are markers of Parkinson disease. Loops of Henle exist in nephrons.

19. A nurse is caring for a patient who has been diagnosed with leukemia. The nurses most recent assessment reveals the presence of ecchymoseson the patients sacral area and petechiae in her forearms. In addition to informing the patients primary care provider, the nurse should perform what action? A) Initiate measures to prevent venous thromboembolism (VTE). B) Check the patients most recent platelet level. C) Place the patient on protective isolation. D) Ambulate the patient to promote circulatory function.

Ans: B Feedback: The patients signs are suggestive of thrombocytopenia, thus the nurse should check the patients most recent platelet level. VTE is not a risk and this does not constitute a need for isolation. Ambulation and activity may be contraindicated due to the risk of bleeding.

11. A patient undergoing a hip replacement has autologous blood on standby if a transfusion is needed. What is the primary advantage of autologous transfusions? A) Safe transfusion for patients with a history of transfusion reactions B) Prevention of viral infections from another persons blood C) Avoidance of complications in patients with alloantibodies D) Prevention of alloimmunization

Ans: B Feedback: The primary advantage of autologous transfusions is the prevention of viral infections from another persons blood. Other secondary advantages include safe transfusion for patients with a history of transfusion reactions, prevention of alloimmunization, and avoidance of complications in patients with alloantibodies.

2. A patient newly diagnosed with thrombocytopenia is admitted to the medical unit. After the admission assessment, the patient asks the nurse to explain the disease. What should the nurse explain to this patient? A) There could be an attack on the platelets by antibodies. B) There could be decreased production of platelets. C) There could be impaired communication between platelets. D) There could be an autoimmune process causing platelet malfunction.

Ans: B Feedback: Thrombocytopenia can result from a decreased platelet production, increased platelet destruction, or increased consumption of platelets. Impaired platelet communication, antibodies, and autoimmune processes are not typical pathologies.

11.integumentary system, what finding would be most consistent with this platelet level? A patients blood work reveals a platelet level of 17,000/mm3. When inspecting the patients A) Dermatitis B) Petechiae C) Urticaria D) Alopecia

Ans: B Feedback: When the platelet count drops to less than 20,000/mm3, petechiae can appear. Low platelet levels do not normally result in dermatitis, urticaria (hives), or alopecia (hair loss).

5. The nurse is assessing a new patient with complaints of overwhelming fatigue and a sore tongue that is visibly smooth and beefy red. This patient is demonstrating signs and symptoms associated with what form of what hematologic disorder? A) Sickle cell anemia B) Hemophilia C) Megaloblastic anemia D) Thrombocytopenia

Ans: C Feedback: A red, smooth, sore tongue is a symptom associated with megaloblastic anemia. Sickle cell disease, hemophilia, and thrombocytopenia do not have symptoms involving the tongue.

31 A client with several chronic health problems has been newly diagnosed with a qualitative platelet defect. What component of the patients previous medication regimen may have contributed to the development of this disorder? A) Calcium carbonate B) Vitamin B12 C) Aspirin D) Vitamin D

Ans: C Feedback: Aspirin may induce a platelet disorder. Even small amounts of aspirin reduce normal platelet aggregation, and the prolonged bleeding time lasts for several days after aspirin ingestion. Calcium, vitamin D, and vitamin B12 do not have the potential to induce a platelet defect.

39. Which of the following circumstances would most clearly warrant autologous blood donation? A) The patient has type-O blood. B) The patient has sickle cell disease or a thalassemia. C) The patient has elective surgery pending. D) The patient has hepatitis C.

Ans: C Feedback: Autologous blood donation is useful for many elective surgeries where the potential need for transfusion is high. Type-O blood, hepatitis, sickle cell disease, and thalassemia are not clear indications for autologous donation.

11. An emergency department nurse is triaging a 77-year-old man who presents with uncharacteristic fatigue as well as back and rib pain. The patient denies any recent injuries. The nurse should recognize the need for this patient to be assessed for what health problem? A) Hodgkin disease B) Non-Hodgkin lymphoma C) Multiple myeloma D) Acute thrombocythemia

Ans: C Feedback: Back pain, which is often a presenting symptom in multiple myeloma, should be closely investigated in older patients. The lymphomas and bleeding disorders do not typically present with the primary symptom of back pain or rib pain.

29. A patient with a myelodysplastic syndrome (MDS) is being treated on the medical unit. What assessment finding should prompt the nurse to contact the patients primary care provider? A) The patient is experiencing a frontal lobe headache. B) The patient has an episode of urinary incontinence. C) The patient has an oral temperature of 37.5C (99.5F). D) The patients SpO2 is 91% on room air.

Ans: C Feedback: Because the patient with MDS is at a high risk for infection, any early signs of infection must be reported promptly. The nurse should address each of the listed assessment findings, but none is as direct a threat to the patients immediate health as an infection.

4. A nurse is planning the care of a patient who has been admitted to the medical unit with a diagnosis of multiple myeloma. In the patients care plan, the nurse has identified a diagnosis of Risk for Injury. What pathophysiologic effect of multiple myeloma most contributes to this risk? A) Labyrinthitis B) Left ventricular hypertrophy C) Decreased bone density D) Hypercoagulation

Ans: C Feedback: Clients with multiple myeloma are at risk for pathologic bone fractures secondary to diffuse osteoporosis and osteolytic lesions. Labyrinthitis is uncharacteristic, and patients do not normally experience hypercoagulation or cardiac hypertrophy.

34. A nurse at a long-term care facility is amending the care plan of a resident who has just been diagnosed with essential thrombocythemia (ET). The nurse should anticipate the administration of what medication? A) Dalteparin B) Allopurinol C) Hydroxyurea D) Hydrochlorothiazide

Ans: C Feedback: Hydroxyurea is effective in lowering the platelet count for patients with ET. Dalteparin, allopurinol, and HCTZ do not have this therapeutic effect.

26. After receiving a diagnosis of acute lymphocytic leukemia, (ALL) a patient is visibly distraught, stating, I have no idea where to go from here. How should the nurse prepare to meet this patients psychosocial needs? A) Assess the patients previous experience with the health care system. B) Reassure the patient that treatment will be challenging but successful. C) Assess the patients specific needs for education and support. D) Identify the patients plan of medical care.

Ans: C Feedback: In order to meets the patients needs, the nurse must first identify the specific nature of these needs. According to the nursing process, assessment must precede interventions. The plan of medical care is important, but not central to the provision of support. The patients previous health care is not a primary consideration, and the nurse cannot assure the patient of successful treatment.

15. The nurse on the pediatric unit is caring for a 10-year-old boy with a diagnosis of hemophilia. The nurse knows that a priority nursing diagnosis for a patient with hemophilia is what? A) Hypothermia B) Diarrhea C) Ineffective coping D) Imbalanced nutrition: Less than body requirements

Ans: C Feedback: Most patients with hemophilia are diagnosed as children. They often require assistance in coping with the condition because it is chronic, places restrictions on their lives, and is an inherited disorder that can be passed to future generations. Children with hemophilia are not at risk of hypothermia, diarrhea, or imbalanced nutrition.

37. Fresh-frozen plasma (FFP) has been ordered for a hospital patient. Prior to administration of this blood product, the nurse should prioritize what patient education? A) Infection risks associated with FFP administration B) Physiologic functions of plasma C) Signs and symptoms of a transfusion reaction D) Strategies for managing transfusion-associated anxiety

Ans: C Feedback: Patients should be educated about signs and symptoms of transfusion reactions prior to administration of any blood product. In most cases, this is priority over education relating to infection. Anxiety may be an issue for some patients, but transfusion reactions are a possibility for all patients. Teaching about the functions of plasma is not likely a high priority.

24. A patient has been scheduled for a bone marrow biopsy and admits to the nurse that she is worried about the pain involved with the procedure. What patient education is most accurate? A) Youll be given painkillers before the test, so there wont likely be any pain? B) Youll feel some pain when the needle enters your skin, but none when the needle enters the bone because of the absence of nerves in bone. C) Most people feel some brief, sharp pain when the needle enters the bone. D) Ill be there with you, and Ill try to help you keep your mind off the pain.

Ans: C Feedback: Patients typically feel a pressure sensation as the needle is advanced into position. The actual aspiration always causes sharp, but brief pain, resulting from the suction exerted as the marrow is aspirated into the syringe; the patient should be warned about this. Stating, Ill try to help you keep your mind off the pain may increase the patients fears of pain, because this does not help the patient know what to expect.

7. A 77-year-old male is admitted to a unit with a suspected diagnosis of acute myeloid leukemia (AML). When planning this patients care, the nurse should be aware of what epidemiologic fact? A) Early diagnosis is associated with good outcomes. B) Five-year survival for older adults is approximately 50%. C) Five-year survival for patients over 75 years old is less than 2%. D) Survival rates are wholly dependent on the patients pre-illness level of health.

Ans: C Feedback: The 5-year survival rate for patients with AML who are 50 years of age or younger is 43%; it drops to 19% for those between 50 and 64 years, and drops to1.6% for those older than 75 years. Early diagnosis is beneficial, but is nonetheless not associated with good outcomes or high survival rates. Preillness health is significant, but not the most important variable.

4. A nurse is providing education to a patient with iron deficiency anemia who has been prescribed iron supplements. What should the nurse include in health education? A) Take the iron with dairy products to enhance absorption. B) Increase the intake of vitamin E to enhance absorption. C) Iron will cause the stools to darken in color. D) Limit foods high in fiber due to the risk for diarrhea.

Ans: C Feedback: The nurse will inform the patient that iron will cause the stools to become dark in color. Iron should be taken on an empty stomach, as its absorption is affected by food, especially dairy products. Patients should be instructed to increase their intake of vitamin C to enhance iron absorption. Foods high in fiber should be consumed to minimize problems with constipation, a common side effect associated with iron therapy.

21. A nurse is planning the care of a patient with a diagnosis of sickle cell disease who has been admitted for the treatment of an acute vaso-occlusive crisis. What nursing diagnosis should the nurse prioritize in the patients plan of care? A) Risk for disuse syndrome related to ineffective peripheral circulation B) Functional urinary incontinence related to urethral occlusion C) Ineffective tissue perfusion related to thrombosis D) Ineffective thermoregulation related to hypothalamic dysfunction

Ans: C Feedback: There are multiple potential complications of sickle cell disease and sickle cell crises. Central among these, however, is the risk of thrombosis and consequent lack of tissue perfusion. Sickle cell crises are not normally accompanied by impaired thermoregulation or genitourinary complications. Risk for disuse syndrome is not associated with the effects of acute vaso-occlusive crisis.

28. A nurse is a long-term care facility is admitting a new resident who has a bleeding disorder. When planning this residents care, the nurse should include which of the following? A) Housing the resident in a private room B) Implementing a passive ROM program to compensate for activity limitation C) Implementing of a plan for fall prevention D) Providing the patient with a high-fiber diet

Ans: C Feedback: To prevent bleeding episodes, the nurse should ensure that an older adult with a bleeding disorder does not suffer a fall. Activity limitation is not necessarily required, however. A private room is not necessary and there is no reason to increase fiber intake.

3. A patient has come to the OB/GYN clinic due to recent heavy menstrual flow. Because of the patients consequent increase in RBC production, the nurse knows that the patient may need to increase her daily intake of what substance? A) Vitamin E B) Vitamin D C) Iron D) Magnesium

Ans: C Feedback: To replace blood loss, the rate of red cell production increases. Iron is incorporated into hemoglobin. Vitamins E and D and magnesium do not need to be increased when RBC production is increased.

12. A nurse is admitting a patient with immune thrombocytopenic purpura to the unit. In completing the admission assessment, the nurse must be alert for what medications that potentially alter platelet function? Select all that apply. A) Antihypertensives B) Penicillins C) Sulfa-containing medications D) Aspirin-based drugs E) NSAIDs

Ans: C, D, E Feedback: The nurse must be alert for sulfa-containing medications and others that alter platelet function (e.g., aspirin-based or other NSAIDs). Antihypertensive drugs and the penicillins do not alter platelet function.

18. A patient has been living with a diagnosis of anemia for several years and has experienced recent declines in her hemoglobin levels despite active treatment. What assessment finding would signal complications of anemia? A) Venous ulcers and visual disturbances B) Fever and signs of hyperkalemia C) Epistaxis and gastroesophageal reflux D) Ascites and peripheral edema

Ans: D Feedback: A significant complication of anemia is heart failure from chronic diminished blood volume and the hearts compensatory effort to increase cardiac output. Patients with anemia should be assessed for signs and symptoms of heart failure, including ascites and peripheral edema. None of the other listed signs and symptoms is characteristic of heart failure.

35. A patient with Von Willebrand disease (vWD) has experienced recent changes in bowel function that suggest the need for a screening colonoscopy. What intervention should be performed in anticipation of this procedure? A) The patient should not undergo the normal bowel cleansing protocol prior to the procedure. B) The patient should receive a unit of fresh-frozen plasma 48 hours before the procedure. C) The patient should be admitted to the surgical unit on the day before the procedure. D) The patient should be given necessary clotting factors before the procedure.

Ans: D Feedback: A goal of treating vWD is to replace the deficient protein (e.g., vWF or factor VIII) prior to an invasive procedure to prevent subsequent bleeding. Bowel cleansing is not contraindicated and FFP does not reduce the patients risk of bleeding. There may or may not be a need for preprocedure hospital admission.

40. A nurse is caring for patient whose diagnosis of multiple myeloma is being treated with bortezomib. The nurse should assess for what adverse effect of this treatment? A) Stomatitis B) Nephropathy C) Cognitive changes D) Peripheral neuropathy

Ans: D Feedback: A significant toxicity associated with the use of bortezomib for multiple myeloma is peripheral neuropathy. Stomatitis, cognitive changes, and nephropathy are not noted to be adverse effects of this medication.

3. A critical care nurse is caring for a patient with autoimmune hemolytic anemia. The patient is not responding to conservative treatments, and his condition is now becoming life threatening. The nurse is aware that a treatment option in this case may include what? A) Hepatectomy B) Vitamin K administration C) Platelet transfusion D) Splenectomy

Ans: D Feedback: A splenectomy may be the course of treatment if autoimmune hemolytic anemia does not respond to conservative treatment. Vitamin K administration is treatment for vitamin K deficiency and does not resolve anemia. Platelet transfusion may be the course of treatment for some bleeding disorders. Hepatectomy would not help the patient.

17. Diagnostic testing has resulted in a diagnosis of acute myeloid leukemia (AML) in an adult patient who is otherwise healthy. The patient and the care team have collaborated and the patient will soon begin induction therapy. The nurse should prepare the patient for which of the following? A) Daily treatment with targeted therapy medications B) Radiation therapy on a daily basis C) Hematopoietic stem cell transplantation D) An aggressive course of chemotherapy

Ans: D Feedback: Attempts are made to achieve remission of AML by the aggressive administration of chemotherapy, called induction therapy, which usually requires hospitalization for several weeks. Induction therapy is not synonymous with radiation, stem cell transplantation, or targeted therapies.

31. A clinic patient is being treated for polycythemia vera and the nurse is providing health education. What practice should the nurse recommend in order to prevent the complications of this health problem? A) Avoiding natural sources of vitamin K B) Avoiding altitudes of 1500 feet (457 meters) C) Performing active range of motion exercises daily D) Avoiding tight and restrictive clothing on the legs

Ans: D Feedback: Because of the risk of DVT, patients with polycythemia vera should avoid tight and restrictive clothing. There is no need to avoid foods with vitamin K or to avoid higher altitudes. Activity levels should be maintained, but there is no specific need for ROM exercises.

17. An adult patient has been diagnosed with iron-deficiency anemia. What nursing diagnosis is most likely to apply to this patients health status? A) Risk for deficient fluid volume related to impaired erythropoiesis B) Risk for infection related to tissue hypoxia C) Acute pain related to uncontrolled hemolysis D) Fatigue related to decreased oxygen-carrying capacity

Ans: D Feedback: Fatigue is the major assessment finding common to all forms of anemia. Anemia does not normally result in acute pain or fluid deficit. The patient may have an increased risk of infection due to impaired immune function, but fatigue is more likely.

12. A home health nurse is caring for a patient with multiple myeloma. Which of the following interventions should the nurse prioritize when addressing the patients severe bone pain? A) Implementing distraction techniques B) Educating the patient about the effective use of hot and cold packs C) Teaching the patient to use NSAIDs effectively D) Helping the patient manage the opioid analgesic regimen

Ans: D Feedback: For severe pain resulting from multiple myeloma, opioids are likely necessary. NSAIDs would likely be ineffective and are associated with significant adverse effects. Hot and cold packs as well as distraction would be insufficient for severe pain.

14. The nurse educating a patient with anemia is describing the process of RBC production. When the patients kidneys sense a low level of oxygen in circulating blood, what physiologic response is initiated? A) Increased stem cell synthesis B) Decreased respiratory rate C) Arterial vasoconstriction D) Increased production of erythropoietin

Ans: D Feedback: If the kidney detects low levels of oxygen, as occurs when fewer red cells are available to bind oxygen (i.e., anemia), erythropoietin levels increase. The body does not compensate with vasoconstriction, decreased respiration, or increased stem cell activity.

8. A 35-year-old male is admitted to the hospital complaining of severe headaches, vomiting, and testicular pain. His blood work shows reduced numbers of platelets, leukocytes, and erythrocytes, with a high proportion of immature cells. The nurse caring for this patient suspects a diagnosis of what? A) AML B) CML C) MDS D) ALL

Ans: D Feedback: In acute lymphocytic leukemia (ALL), manifestations of leukemic cell infiltration into other organs are more common than with other forms of leukemia, and include pain from an enlarged liver or spleen, as well as bone pain. The central nervous system is frequently a site for leukemic cells; thus, patients may exhibit headache and vomiting because of meningeal involvement. Other extranodal sites include the testes and breasts. This particular presentation is not closely associated with acute myeloid leukemia (AML), chronic myeloid leukemia (CML), or myelodysplastic syndromes (MDS).

8. The nurse is caring for a patient who has developed scar tissue in many of the areas that normally produce blood cells. What organs can become active in blood cell production by the process of extramedullary hematopoiesis? A) Spleen and kidneys B) Kidneys and pancreas C) Pancreas and liver D) Liver and spleen

Ans: D Feedback: In adults with disease that causes marrow destruction, fibrosis, or scarring, the liver and spleen can also resume production of blood cells by a process known as extramedullary hematopoiesis. The kidneys and pancreas do not produce blood cells for the body.

8. A patient comes into the clinic complaining of fatigue. Blood work shows an increased bilirubin concentration and an increased reticulocyte count. What would the nurse suspect the patient has? A) A hypoproliferative anemia B) A leukemia C) Thrombocytopenia D) A hemolytic anemia

Ans: D Feedback: In hemolytic anemias, premature destruction of erythrocytes results in the liberation of hemoglobin from the erythrocytes into the plasma; the released hemoglobin is converted in large part to bilirubin, and therefore the bilirubin concentration rises. The increased erythrocyte destruction leads to tissue hypoxia, which in turn stimulates erythropoietin production. This increased production is reflected in an increased reticulocyte count as the bone marrow responds to the loss of erythrocytes. Hypoproliferative anemias, leukemia, and thrombocytopenia lack this pathology and presentation.

35. A patient lives with a diagnosis of sickle cell anemia and receives frequent blood transfusions. The nurse should recognize the patients consequent risk of what complication of treatment? A) Hypovolemia B) Vitamin B12 deficiency C) Thrombocytopenia D) Iron overload

Ans: D Feedback: Patients with chronic transfusion requirements can quickly acquire more iron than they can use, leading to iron overload. These individuals are not at risk for hypovolemia and there is no consequent risk for low platelet or vitamin B12 levels.

22. A patient is being treated on the medical unit for a sickle cell crisis. The nurses most recent assessment reveals an oral temperature of 100.5F and a new onset of fine crackles on lung auscultation. What is the nurses most appropriate action? A) Apply supplementary oxygen by nasal cannula. B) Administer bronchodilators by nebulizer. C) Liaise with the respiratory therapist and consider high-flow oxygen. D) Inform the primary care provider that the patient may have an infection.

Ans: D Feedback: Patients with sickle cell disease are highly susceptible to infection,thus any early signs of infection should be reported promptly. There is no evidence of respiratory distress, so oxygen therapy and bronchodilators are not indicated.

13. A patient, 25 years of age, comes to the emergency department complaining of excessive bleeding from a cut sustained when cleaning a knife. Blood work shows a prolonged PT but a vitamin K deficiency is ruled out. When assessing the patient, areas of ecchymosis are noted on other areas of the body. Which of the following is the most plausible cause of the patients signs and symptoms? A) Lymphoma B) Leukemia C) Hemophilia D) Hepatic dysfunction

Ans: D Feedback: Prolongation of the PT, unless it is caused by vitamin K deficiency, may indicate severe hepatic dysfunction. The majority of hemophiliacs are diagnosed as children. The scenario does not describe signs or symptoms of lymphoma or leukemia.

38. An intensive care nurse is aware of the need to identify patients who may be at risk of developing disseminated intravascular coagulation (DIC). Which of the following ICU patients most likely faces the highest risk of DIC? A) A patient with extensive burns B) A patient who has a diagnosis of acute respiratory distress syndrome C) A patient who suffered multiple trauma in a workplace accident D) A patient who is being treated for septic shock

Ans: D Feedback: Sepsis is a common cause of DIC. A wide variety of acute illnesses can precipitate DIC, but sepsis is specifically identified as a cause.

27. A patient has completed the full course of treatment for acute lymphocytic leukemia (ALL) and has failed to respond appreciably. When preparing for the patients subsequent care, the nurse should perform what action? A) Arrange a meeting between the patients family and the hospital chaplain. B) Assess the factors underlying the patients failure to adhere to the treatment regimen. C) Encourage the patient to vigorously pursue complementary and alternative medicine (CAM). D) Identify the patients specific wishes around end-of-life care.

Ans: D Feedback: Should the patient not respond to therapy, it is important to identify and respect the patients choices about treatment, including measures to prolong life and other end-of-life measures. The patient may or may not be open to pursuing CAM. Unsuccessful treatment is not necessarily the result of failure to adhere to the treatment plan. Assessment should precede meetings with a chaplain, which may or may not be beneficial to the patient and congruent with the familys belief system.

23. A patients diagnosis of atrial fibrillation has prompted the primary care provider to prescribe warfarin (Coumadin), an anticoagulant. When assessing the therapeutic response to this medication, what is the nurses most appropriate action? A) Assess for signs of myelosuppression. B) Review the patients platelet level. C) Assess the patients capillary refill time. D) Review the patients international normalized ratio (INR).

Ans: D Feedback: The INR and aPTT serve as useful screening tools for evaluating a patients clotting ability and to monitor the therapeutic effectiveness of anticoagulant medications. The patients platelet level is not normally used as a short-term indicator of anticoagulation effectiveness. Assessing the patient for signs of myelosuppression and capillary refill time does not address the effectiveness of anticoagulants.

32. A clinic nurse is working with a patient who has a long-standing diagnosis of polycythemia vera. How can the nurse best gauge the course of the patients disease? A) Document the color of the patients palms and face during each visit. B) Follow the patients erythrocyte sedimentation rate over time. C) Document the patients response to erythropoietin injections. D) Follow the trends of the patients hematocrit.

Ans: D Feedback: The course of polycythemia vera can be best ascertained by monitoring the patients hematocrit, which should remain below 45%. Erythropoietin injections would exacerbate the condition. Skin tone should be observed, but is a subjective assessment finding. The patients ESR is not relevant to the course of the disease.

33. An interdisciplinary team has been commissioned to create policies and procedures aimed at preventing acute hemolytic transfusion reactions. What action has the greatest potential to reduce the risk of this transfusion reaction? A) Ensure that blood components are never infused at a rate greater than 125 mL/hr. B) Administer prophylactic antihistamines prior to all blood transfusions. C) Establish baseline vital signs for all patients receiving transfusions. D) Be vigilant in identifying the patient and the blood component.

Ans: D Feedback: The most common causes of acute hemolytic reaction are errors in blood component labeling and patient identification that result in the administration of an ABO-incompatible transfusion. Actions to address these causes are necessary in all health care settings. Prophylactic antihistamines are not normally administered, and would not prevent acute hemolytic reactions. Similarly, baseline vital signs and slow administration will not prevent this reaction.

14. A patient with non-Hodgkins lymphoma is receiving information from the oncology nurse. The patient asks the nurse why she should stop drinking and smoking and stay out of the sun. What would be the nurses best response? A) Everyone should do these things because theyre health promotion activities that apply to everyone. B) You dont want to develop a second cancer, do you? C) You need to do this just to be on the safe side. D) Its important to reduce other factors that increase the risk of second cancers.

Ans: D Feedback: The nurse should encourage patients to reduce other factors that increase the risk of developing second cancers, such as use of tobacco and alcohol and exposure to environmental carcinogens and excessive sunlight. The other options do not answer the patients question, and also make light of the patients question.

39. A patient is being treated for DIC and the nurse has prioritized the nursing diagnosis of Risk for Deficient Fluid Volume Related to Bleeding. How can the nurse best determine if goals of care relating to this diagnosis are being met? A) Assess for edema. B) Assess skin integrity frequently. C) Assess the patients level of consciousness frequently. D) Closely monitor intake and output.

Ans: D Feedback: The patient with DIC is at a high risk of deficient fluid volume. The nurse can best gauge the effectiveness of care by closely monitoring the patients intake and output. Each of the other assessments is a necessary element of care, but none addresses fluid balance as directly as close monitoring of intake and output.

p590-602 10. A patients wound has begun to heal and the blood clot which formed is no longer necessary. When a blood clot is no longer needed, the fibrinogen and fibrin will be digested by which of the following? A) Plasminogen B) Thrombin C) Prothrombin D) Plasmin

Ans: D Feedback: The substance plasminogen is required to lyse (break down) the fibrin. Plasminogen, which is present in all body fluids, circulates with fibrinogen and is therefore incorporated into the fibrin clot as it forms. When the clot is no longer needed (e.g., after an injured blood vessel has healed), the plasminogen is activated to form plasmin. Plasmin digests the fibrinogen and fibrin. Prothrombin is converted to thrombin, which in turn catalyzes the conversion of fibrinogen to fibrin so a clot can form.

40. A patient with a pulmonary embolism is being treated with a heparin infusion. What diagnostic finding suggests to the nurse that treatment is effective? A) The patients PT is within reference ranges. B) Arterial blood sampling tests positive for the presence of factor XIII. C) The patients platelet level is below 100,000/mm3. D) The patients activated partial thromboplastin time (aPTT) is 1.5 to 2.5 times the control value.

Ans: D Feedback: The therapeutic effect of heparin is monitored by serial measurements of the aPTT; the dose is adjusted to maintain the range at 1.5 to 2.5 times the laboratory control. Heparin dosing is not determined on the basis of platelet levels, the presence or absence of clotting factors, or PT levels.

32.4- 4. The nurse is planning the care of a patient with a nutritional deficit and a diagnosis of megaloblastic anemia. The nurse should recognize that this patients health problem is due to what? A) Production of inadequate quantities of RBCs B) Premature release of immature RBCs C) Injury to the RBCs in circulation D) Abnormalities in the structure and function RBCs

Ans: D Feedback: Vitamin B12 and folic acid deficiencies are characterized by the production of abnormally large erythrocytes called megaloblasts. Because these cells are abnormal, many are sequestered (trapped) while still in the bone marrow, and their rate of release is decreased. Some of these cells actually die in the marrow before they can be released into the circulation. This results in megaloblastic anemia. This pathologic process does not involve inadequate production, premature release, or injury to existing RBCs.

18. A nurse is educating a patient about the role of B lymphocytes. The nurses description will include which of the following physiologic processes? A) Stem cell differentiation B) Cytokine production C) Phagocytosis D) Antibody production

Ans: D Feedback:B lymphocytes are capable of differentiating into plasma cells. Plasma cells, in turn, produce antibodies. Cytokines are produced by NK cells. Stem cell differentiation greatly precedes B lymphocyte production.

p649- Autotransfusion, or autologous transfusion, involves removing whole blood from a person and transfusing that blood back into the same person. This avoids problems of incompatibility, allergic reactions, and disease transmission. Methods of autotransfusion include:

Autologous donation or elective phlebotomy (predeposit transfusion). A person donates blood before a planned surgical procedure. The blood can be frozen and stored for up to 10 years. Usually the blood is stored without being frozen and given to the person within a few weeks of donation. This technique is especially beneficial to the patient with a rare blood type or for any patient who may be expected to need limited blood product support during a major surgical procedure (e.g., elective orthopedic surgery).


Ensembles d'études connexes

ECON 211 Ch.3 Dynamic Study Module

View Set

chapter 15 senses; BIO 135: Basic Anatomy/Physiology w/Lab (4226_25ZA); unit 4 test review

View Set

Chapter 11- Appraisal Methods- California Real Estate Principles- 14th edition.

View Set

The Battle of the Little Bighorn

View Set

Comm 1100 Chapter 13: small group communication

View Set

Chapter 6 Quiz Computer Networking Fundamentals

View Set

Section 10 Losing your Driving Privileges

View Set